edited by
244 views

1 Answer

0 votes
0 votes
$\text{Both matrices $A$ and $B$ belong to the set of real matrices of size $3\times 3$}$

$\text{Suppose, $AB-BA = C$ and eigen values of C are $\lambda_1$,$\lambda_2$ and $\lambda_3$}$

$\text{Trace($(AB-BA)^3$)= Trace($C^3)$}$

$= \lambda_1^3 +\lambda_2^3 + \lambda_3^3 = (\lambda_1+\lambda_2 +\lambda_3)(\lambda_1^2+\lambda_2^2 + \lambda_3^2  – \lambda_1\lambda_2-\lambda_2\lambda_3-\lambda_3\lambda_1) + 3 \lambda_1 \lambda_2 \lambda_3$

$\text{Here, $\lambda_1+\lambda_2 +\lambda_3$ = Trace$(AB-BA)=$ Trace$(AB)-$Trace$(BA)=0$}$ $(\because Trace(AB) = Trace(BA) )$

$\text{So, Trace($C^3)= 3 \lambda_1 \lambda_2 \lambda_3 = 3 $ det$(C)$}$

$\text{Hence, $\frac{Trace((AB-BA)^3)}{3} = det(AB-BA)$}$
Answer:

Related questions

0 votes
0 votes
0 answers
1
soujanyareddy13 asked Aug 28, 2020
314 views
True/False Question :There exists no monotone function $f:\mathbb{R}\rightarrow \mathbb{R}$ which is discontinuous at every rational number.
0 votes
0 votes
0 answers
3
soujanyareddy13 asked Aug 28, 2020
178 views
True/False Question :Let $\left \{ a_{n} \right \}^{\infty }_{n=1}$ be a bounded sequence of positive real numbers. Then: $$\underset{n\rightarrow\infty }{lim sup}\:\frac...